Series and parallel circuits Flashcards

1
Q

what is a series circuit?

A

a complete circuit with more than 1 load and only one pathway.

How well did you know this?
1
Not at all
2
3
4
5
Perfectly
2
Q

voltage law

A

total of all voltage drops is equal to the applied voltage. basically all the voltage is used.

How well did you know this?
1
Not at all
2
3
4
5
Perfectly
3
Q

what is another name for voltage drop

A

IR drop because the voltage drop is equal to the amps*resistance

How well did you know this?
1
Not at all
2
3
4
5
Perfectly
4
Q

its important to do what before measuring resistance?

A

disconnect the load from the circuit

How well did you know this?
1
Not at all
2
3
4
5
Perfectly
5
Q

why measure voltage drop instead of resistance

A

1) you can only test resistance with a disconnected component and resistance changes with current.
2) voltage drop is dynamic, takes heat into consideration.

How well did you know this?
1
Not at all
2
3
4
5
Perfectly
6
Q

what are some automotive applications of voltage drop

A

blower motor, dash lights

How well did you know this?
1
Not at all
2
3
4
5
Perfectly
7
Q

Parallel circuits

A

a parallel circuit is a complete circuit with more than one path for the current to follow.

How well did you know this?
1
Not at all
2
3
4
5
Perfectly
8
Q

does an open in one branch of a parallel circuit cause the other branches to stop working

A

no

Counterintuitively the resistance increases and the current will decrease. Its like the electricity knows what loads are ahead in the circuit and sends less current for less load.

1/Rt = 1/4 + 1/8 + 1/16
Rt = 2.28
I = 12/2.28
I = 5.26amps
1/Rt = 1/4 + 1/8
Rt = 2.66
I = 12/2.66
I = 4.51 amps
How well did you know this?
1
Not at all
2
3
4
5
Perfectly
9
Q

parallel circuit law 1 current

A

not all of the current flows through each branch.

How well did you know this?
1
Not at all
2
3
4
5
Perfectly
10
Q

parallel circut law 2 voltage

A

voltage is the same for each branch

How well did you know this?
1
Not at all
2
3
4
5
Perfectly
11
Q

parallel circuit law 3

A

the sum of the current in each leg equals the total current

How well did you know this?
1
Not at all
2
3
4
5
Perfectly
12
Q

how can you measure the total current in a parallel circuit?

A

treat each leg as a simple circuit. the total current is the sum of all the legs.

How well did you know this?
1
Not at all
2
3
4
5
Perfectly
13
Q

the total resistance of a parallel circuit will be more/less than the smallest resistance in any leg?

A

less

How well did you know this?
1
Not at all
2
3
4
5
Perfectly
14
Q

what is kirchoffs voltage law?

A

the total of all voltage drops is equal to applied voltage.

How well did you know this?
1
Not at all
2
3
4
5
Perfectly
15
Q

what would current do if voltage was doubled in a circuit?

A
E=IR
I=E/R
6=12/2
12=24/2
current doubles
How well did you know this?
1
Not at all
2
3
4
5
Perfectly
16
Q

what would current do if resistance was doubled in a circuit?

A

I=E/R
6=12/2
3=12/4
current would be halved

17
Q

What is the forumula for voltage drop?

A

Ohm’s law
E=IR

IR drop

18
Q

Why is the total resistance of a parallel circuit less than the smallest resistance?

A

1/Rt = 1/R1 + 1/R2
I don’t really get it. More answers here:https://physics.stackexchange.com/questions/269492/why-is-the-resistance-in-a-parallel-circuit-smaller-than-the-resistance-in-a-ser

19
Q

Why are parallel circuits used instead of series circuits in most automotive applications?

A

If one leg of a parallel circuit looses continuity the other legs will still work. Dont have a great understanding about this one, need to do more digging.

20
Q

Kirchoffs current law

A

the current flowing into a junction is the same as the current flowing out of a junction

21
Q

What would be the efect of an open circuit in one leg of a parallel portion of a series-parallel circuit?

A

only the leg with the open would be effected.

22
Q

What would be the effect of an open circuit in the series portion of a series-parallel circuit?

A

whole thing would stop working

23
Q

the amperage in a series circuit is the same or different ?

A

the same everywhere

24
Q

the sum of votage drops equals

A

applied voltage

25
Q

what is the formula for finding current

A

E=IR

I=E/R

26
Q

A series circuit has three reistsors of 4ohms each. The voltage drop across each resistor is 4v. Tech A says the source voltage is 12v. Tech B says the total resitance is 18ohms. Which is correct?

A

kirchoffs voltage law. the applied voltage is equal to the sum of all voltage drops. 4v*3=12v so source voltage is 12v. Total resistance in a series circuit is the sum of all resistances so the total resistance is 12ohms not 18. TECH A.

27
Q

a series circuit has two 10ohm bulbs. a third bulb is added in series. Tech A says that the three bulbs will be dimmer than when only two bulbs were in the circuit. Tech B says that the current will increase. Which tech is correct?

A

Three bulbs will be dimmer than two because there is more resistance to flow and therefore less current.

I=E/R

  1. 6amps = 12v/20ohms
  2. 4amps = 12v/30ohms
28
Q

Tech A says that the sum of the voltage drops in a series circuit should equal the source voltage. Tech B says that the current varies depending on the value of resitance in a series circuit.

A

Both techs are correct.

Kirchoffs voltage law states that applied voltage is equal to the sum of voltage drops. Therefore Tech A is correct.

Increased resistance decreases flow (current)
I=E/R
0.6amps = 12v/20ohms 
0.4amps = 12v/30ohms
Therefore Tech B is correct.
29
Q

Two bulbs are connected in parallel to a 12v battery. One has 6ohm resitance the other 2ohm resistance. Which bulb will be brighter?

A

Resistance decreases flow (current) and current is brightness. Therefore the 2ohm bulb will be brighter.

30
Q

Calculate the total resistance and current in a parallel circuit with three resistances, 4ohms, 8ohms, and 16ohms. Assume 12v?

A
1/Rt = 1/4 + 1/8 + 1/16
1/Rt = .4375
Rt = 1/.4375
Rt = 2.28ohms

E=IR
I=E/R
I=12/2.28
I = 5.26amps

31
Q

A vehicle has four parking lights in parallel, one of them blows. Tech A says that this could cause the fuse to blow. Tech B says that the current would decrease.

A

Increased resistance decreases flow (amps). Therefore current would increase. If current increased beyond the rating of the fuse the fuse could blow. Therefore Tech A is correct.

1/Rt = 1/2 + 1/2+ 1/2 +1/2
1/Rt = 2
Rt = 1/2
Rt = 0.5ohms
1/Rt = 1.5
Rt = 1/1.5
Rt = 0.6ohms